Difference between revisions of "2003 AMC 10B Problems/Problem 25"

(Answer to Problem 25)
 
Line 6: Line 6:
  
 
==Solution==
 
==Solution==
 
+
===Solution 1===
 
To test if a number is divisible by three, you add up the digits of the number. If the sum is divisible by three, then the original number is a multiple of three. If the sum is too large, you can repeat the process until you can tell whether it is a multiple of three. This is the basis for the solution. Since the last two digits are <math> 23 </math>, the sum of the digits is <math> 2+3 = 5 </math> (therefore it is not divisible by three). However certain numbers can be added to make the sum of the digits a multiple of three.
 
To test if a number is divisible by three, you add up the digits of the number. If the sum is divisible by three, then the original number is a multiple of three. If the sum is too large, you can repeat the process until you can tell whether it is a multiple of three. This is the basis for the solution. Since the last two digits are <math> 23 </math>, the sum of the digits is <math> 2+3 = 5 </math> (therefore it is not divisible by three). However certain numbers can be added to make the sum of the digits a multiple of three.
  
Line 32: Line 32:
 
And finally, we add the number of elements in each set.
 
And finally, we add the number of elements in each set.
  
<cmath> 1+4+7+9+6+3 = \boxed{\mathrm{(B)}\ 30} </cmath>
+
<cmath> 1+4+7+9+6+3 = \boxed{\textbf{(B)}\ 30} </cmath>
 
   
 
   
 +
===Solution 2===
 +
A number divisible by <math>3</math> has all its digits add to a multiple of <math>3.</math> The last two digits are <math>2</math> and <math>3</math> and add up to <math>5 \equiv 2\ (\text{mod}\ 3).</math> Therefore the first two digits must add up to <math>1\ (\text{mod}\ 3).</math> <math>4</math> digits (including <math>0</math>) are <math>0\ (\text{mod}\ 3),</math> <math>3</math> are <math>1\ (\text{mod}\ 3),</math> and <math>3</math> are <math>2\ (\text{mod}\ 3).</math> The following combinations are equivalent to <math>1\ (\text{mod}\ 3)</math>:
 +
 +
<cmath> 0\ (\text{mod}\ 3)+1\ (\text{mod}\ 3) \equiv 1\ (\text{mod}\ 3)+0\ (\text{mod}\ 3) \equiv 2\ (\text{mod}\ 3) +2\ (\text{mod}\ 3)</cmath>
 +
 +
Let the first term in each combination represent the thousands digit and the second term represent the hundreds digit. We can use this to find the total amount of four-digit numbers.
 +
 +
<cmath>3\cdot3 + 3\cdot4 + 3\cdot3 = 9 + 12 + 9 = \boxed{\textbf{(B)}\ 30}</cmath>
 +
 
== See also ==
 
== See also ==
 
{{AMC10 box|year=2002|num-b=24|after=Last problem|ab=A}}
 
{{AMC10 box|year=2002|num-b=24|after=Last problem|ab=A}}
  
 
[[Category:Counting and Probability]]
 
[[Category:Counting and Probability]]

Revision as of 21:49, 26 November 2011

Problem

How many distinct four-digit numbers are divisible by $3$ and have $23$ as their last two digits?

$\textbf{(A) }27\qquad\textbf{(B) }30\qquad\textbf{(C) }33\qquad\textbf{(D) }81\qquad\textbf{(E) }90$

Solution

Solution 1

To test if a number is divisible by three, you add up the digits of the number. If the sum is divisible by three, then the original number is a multiple of three. If the sum is too large, you can repeat the process until you can tell whether it is a multiple of three. This is the basis for the solution. Since the last two digits are $23$, the sum of the digits is $2+3 = 5$ (therefore it is not divisible by three). However certain numbers can be added to make the sum of the digits a multiple of three.

$5+1 = 6$,

$5+4 = 9$, and so on.

However since the largest four-digit number ending with $23$ is $9923$, the maximum sum is

$5+18 = 23$.

Using that process we can fairly quickly compile a list of the sum of the first two digits of the number.

\[\{1, 4, 7, 10, 13, 16\}\]

Now we find all the two-digit numbers that have any of the sums shown above. We can do this by listing all the two digit numbers $xy$ in separate cases.

\[I. x+y = 1, \{10\} = 1\] \[II. x+y = 4, \{13, 22, 31, 40\} = 4\] \[III. x+y = 7, \{16, 25, 34, 43, 52, 61, 70\} = 7\] \[IV. x+y = 10, \{19, 28, 37, 46, 55, 64, 73, 82, 91\} = 9\] \[V. x+y = 13, \{49, 58, 67, 76, 85, 94\} = 6\] \[VI. x+y = 16, \{79, 88, 97\} = 3\]

And finally, we add the number of elements in each set.

\[1+4+7+9+6+3 = \boxed{\textbf{(B)}\ 30}\]

Solution 2

A number divisible by $3$ has all its digits add to a multiple of $3.$ The last two digits are $2$ and $3$ and add up to $5 \equiv 2\ (\text{mod}\ 3).$ Therefore the first two digits must add up to $1\ (\text{mod}\ 3).$ $4$ digits (including $0$) are $0\ (\text{mod}\ 3),$ $3$ are $1\ (\text{mod}\ 3),$ and $3$ are $2\ (\text{mod}\ 3).$ The following combinations are equivalent to $1\ (\text{mod}\ 3)$:

\[0\ (\text{mod}\ 3)+1\ (\text{mod}\ 3) \equiv 1\ (\text{mod}\ 3)+0\ (\text{mod}\ 3) \equiv 2\ (\text{mod}\ 3) +2\ (\text{mod}\ 3)\]

Let the first term in each combination represent the thousands digit and the second term represent the hundreds digit. We can use this to find the total amount of four-digit numbers.

\[3\cdot3 + 3\cdot4 + 3\cdot3 = 9 + 12 + 9 = \boxed{\textbf{(B)}\ 30}\]

See also

2002 AMC 10A (ProblemsAnswer KeyResources)
Preceded by
Problem 24
Followed by
Last problem
1 2 3 4 5 6 7 8 9 10 11 12 13 14 15 16 17 18 19 20 21 22 23 24 25
All AMC 10 Problems and Solutions